The passage most strongly supports the inference that Garber would agree with which one of the following statements?

Nativeguy on July 11 at 04:48PM

How is B correct ?

Could someone explain how B is correct

Reply
Create a free account to read and take part in forum discussions.

Already have an account? log in

Emil-Kunkin on July 17 at 01:08AM

Hi, we learn in the last paragraph the the insanely high prices might not have been irrational, since the owners of the original bulbs could recoup their investment by selling the bulbs descended from the original bulbs. We see this specifically in line 50 onwards.